Calculating Jon's Energy Needs for First Week of Weight Gain Program

  • Thread starter Thread starter 385sk117
  • Start date Start date
  • Tags Tags
    Integration
Click For Summary
Jon's energy needs for the first week of his weight gain program are defined by the function E(t) = 350(80 + 0.15t)^0.8 - 120(80 + 0.15t) calories/day. A user attempted to calculate total energy needs by integrating this function over the interval [0, 7] but obtained a result of 14377 calories instead of the expected 14400. Another participant suggested that since the problem specifies daily calorie needs, it would be more accurate to sum the values for each day from E(0) to E(6) instead of integrating. The discrepancy in results highlights the importance of interpreting the problem correctly, particularly regarding the treatment of discrete versus continuous variables. Accurate calculation methods are crucial for determining Jon's total energy needs effectively.
385sk117
Messages
50
Reaction score
0

Homework Statement



Jon needs to bulk-up for next AFL season. His energy needs t days after starting his weight gain program are given by E(t) = 350(80 + 0.15t)^0.8 - 120(80 + 0.15t) calcories/day. Find Jon's total energy needs over the first week of the program.


Homework Equations





The Attempt at a Solution


I integrated the fuction above and interval was [0, 7] as the question says total energy needs over the first week. The answer i got was 14377 but the answer is 14400.
I think there is some problem with the interval or the value of C which i couldn't find.
Help me please~
 
Physics news on Phys.org
There is no "C" in this problem. What do you mean by that? If it were me, I wouldn't integrate. Since you are give "calories per day" and days are discreet quantities, I would just add E(0)+ E(1)+ E(2)+ E(3)+ E(4)+ E(5)+ E(6). Since E(7) is the calories needed after 7 days, it is not included in the calories consumed in the first 7 days.
 
14378 is the answer i got from your method. And this question was under the heading of Integration so i think i should use integration method to solve it.
 
Question: A clock's minute hand has length 4 and its hour hand has length 3. What is the distance between the tips at the moment when it is increasing most rapidly?(Putnam Exam Question) Answer: Making assumption that both the hands moves at constant angular velocities, the answer is ## \sqrt{7} .## But don't you think this assumption is somewhat doubtful and wrong?

Similar threads

Replies
12
Views
2K
Replies
3
Views
1K
  • · Replies 21 ·
Replies
21
Views
3K
  • · Replies 37 ·
2
Replies
37
Views
7K
Replies
1
Views
2K
  • · Replies 1 ·
Replies
1
Views
7K
Replies
1
Views
3K
  • · Replies 1 ·
Replies
1
Views
2K
  • Sticky
  • · Replies 48 ·
2
Replies
48
Views
67K
Replies
8
Views
4K